LSAT and Law School Admissions Forum

Get expert LSAT preparation and law school admissions advice from PowerScore Test Preparation.

 Administrator
PowerScore Staff
  • PowerScore Staff
  • Posts: 8916
  • Joined: Feb 02, 2011
|
#100951
Complete Question Explanation

Must Be True. The correct answer choice is (E).

Answer choice (A):

Answer choice (B):

Answer choice (C):

Answer choice (D):

Answer choice (E): This is the correct answer choice.
 esther913
  • Posts: 23
  • Joined: Apr 13, 2019
|
#65046
Hi,
I am having a hard time understanding why (E) is the correct answer. The problem is that I cannot see (E) as a paraphrase of the stimulus.

I thought the main point/focus of (E) was different from that of the stimulus and considered (E) as a shell game answer choice. (E) seems to be more focused on whether the approaches are compatible or not, while the conclusion of the stimulus is a suggestion for Cultural anthropologists to “ employ both approaches, and also attend to a third."
Why is (E) not a shell game answer choice? :-?

In addition, I think (E) resembles more of an unstated premise/assumption rather than a paraphrase of the stimulus.
Is it possible for an assumption to be the correct answer choice for a Must Be True/Most Strongly Supported question?

Clarification would be greatly appreciated. :)
Thank you.
 Adam Tyson
PowerScore Staff
  • PowerScore Staff
  • Posts: 5153
  • Joined: Apr 14, 2011
|
#65185
Answer E has nothing to do with paraphrasing the stimulus, esther913. The question isn't about what the stimulus says, but about what inferences can be properly drawn based on what the author said. If we accept the information in the stimulus as true, what does that information suggest?

Answer E is very strongly supported by the stimulus. If the author thinks cultural anthropologists should employ both methods and even add a third one, then the author must believe that the various methods are compatible with each other. That is, he must believe taking one approach does not somehow preclude taking one of the other approaches.

As to your last question, if you see answer E as an unstated premise, or assumption, of the argument, that makes it a perfect answer for a Most Strongly Supported or Must Be True question! That means you see that the argument requires the truth of that answer! That's exactly what you should be looking for - not a paraphrase of the argument, but something true that the argument supports.
 esther913
  • Posts: 23
  • Joined: Apr 13, 2019
|
#65211
Wow :0 Now I understand!
Thank you so much for the detailed explanation. I greatly appreciate your help.

Get the most out of your LSAT Prep Plus subscription.

Analyze and track your performance with our Testing and Analytics Package.